Diễn Đàn MathScopeDiễn Đàn MathScope
  Diễn Đàn MathScope
Ghi Danh Hỏi/Ðáp Community Lịch

Go Back   Diễn Đàn MathScope > Sơ Cấp > Việt Nam và IMO

News & Announcements

Ngoài một số quy định đã được nêu trong phần Quy định của Ghi Danh , mọi người tranh thủ bỏ ra 5 phút để đọc thêm một số Quy định sau để khỏi bị treo nick ở MathScope nhé !

* Nội quy MathScope.Org

* Một số quy định chung !

* Quy định về việc viết bài trong diễn đàn MathScope

* Nếu bạn muốn gia nhập đội ngũ BQT thì vui lòng tham gia tại đây

* Những câu hỏi thường gặp

* Về việc viết bài trong Box Đại học và Sau đại học


Trả lời Gởi Ðề Tài Mới
 
Ðiều Chỉnh Xếp Bài
Old 30-03-2008, 08:14 PM   #1
let
+Thành Viên Danh Dự+
 
let's Avatar
 
Tham gia ngày: Nov 2007
Bài gởi: 168
Thanks: 16
Thanked 42 Times in 25 Posts
VietnamTST 2008

KÌ THI CHỌN HỌC SINH VÀO CÁC ĐỘI TUYỂN QUỐC GIA DỰ THI OLYMPIC QUỐC TẾ NĂM 2008

Bài 1. Trên mặt phẳng, cho góc $xOy $. Xét điểm $M $ thay đổi trên tia $Ox $ và điểm $N $ thay đổi trên tia $Oy $. Kí hiệu $d $ là đường phân giác ngoài của góc $xOy $ và gọi $I $ là giao điểm của $d $ với đường trung trực của đoạn thẳng $MN $. Trên $d $, lấy hai điểm $P, Q $ sao cho $IP = IQ = IM = IN $. Gọi $K $ là giao điểm của các đường thẳng $MQ $ và $NP $.
1/ Chứng minh rằng $K $ luôn nằm trên một đường thẳng cố định, khi $M $ và $N $ thay đổi trên $Ox $ và $Oy $.
2/ Xét các điểm $M, N $ trên các tia $Ox $ và $Oy $ sao cho đường thẳng $d_1 $ vuông góc với $IM $ tại $M $ và đường thẳng $d_2 $ vuông góc với $IN $ tại $N $ đều cắt đường thẳng $d $. Gọi $E, F $ tương ứng là giao điểm của $d_1, d_2 $ với $d $. Chứng minh rằng các đường thẳng $EN, FM $ và $OK $ đồng quy.
Bài 2. Hãy xác định tất cả các số nguyên dương $m $ sao cho tồn tại các đa thức với hệ số thực $P(x), Q(x), R(x, y) $ thỏa mãn điều kiện: Với mọi số thực $a, b $ mà $a^m-b^2=0 $, ta luôn có $P(R(a, b)) = a $ và $Q(R(a, b)) = b $.
Bài 3. Cho số nguyên $n > 3 $. Kí hiệu $T $ là tập hợp gồm $n $ số nguyên dương đầu tiên. Một tập con $S $ của $T $ được gọi là tập khuyết trong $T $ nếu $S $ có tính chất: Tồn tại số nguyên dương $c $ không vượt quá $\frac{n}{2} $ sao cho với $s_1, s_2 $ là hai số bất kì thuộc $S $ ta luôn có $|s_1-s_2|\neq c $. Hỏi tập khuyết trong $T $ có thể có tối đa bao nhiêu phần tử ?
Bài 4. Cho $m, n $ là các số nguyên dương. Chứng minh rằng $(2m + 3)^n + 1 $ chia hết cho $6m $ khi và chỉ khi $3^n + 1 $ chia hết cho $4m $.
Bài 5. Cho $k $ là số thực dương. Cho tam giác nhọn, không cân $ABC $ có $O $ là tâm đường tròn ngoại tiếp và $AD, BE, CF $ là các đường phân giác trong. Trên các tia $AD, BE, CF $ lần lượt lấy các điểm $L, M, N $ sao cho $\frac{AL}{AD}=\frac{BM}{BE}=\frac{CN}{CF}=k $. Kí hiệu $(O_1), (O_2), (O_3) $ tương ứng là đường tròn đi qua $L $ và tiếp xúc với $OA $ tại $A $, đường tròn đi qua $M $ và tiếp xúc với $OB $ tại $B $, đường tròn đi qua $N $ và tiếp xúc với $OC $ tại $C $.
1/ Chứng minh rằng với $k=\frac{1}{2} $, ba đường tròn $(O_1), (O_2), (O_3) $ có đúng hai điểm chung và trọng tâm $G $ của tam giác $ABC $ nằm trên đường thẳng đi qua hai điểm chung đó.
2/ Hãy xác định tất cả các giá trị $k $ để ba dường tròn $(O_1), (O_2), (O_3) $ có đúng hai điểm chung.
Bài 6. Kí hiệu $M $ là tập hợp gồm $2008 $ số nguyên dương đầu tiên. Tô tất cả các số thuộc $M $ bởi ba màu xanh, vàng, đỏ sao cho mỗi số được tô bởi một màu và mỗi màu đều được dùng để tô ít nhất một số.
Xét các tập hợp:
$S_1 = \{(x, y, z)\in M^3 | x, y, z $ có cùng màu và $(x + y + z)\equiv 0 (mod\ 2008)\} $;
$S_2 = \{(x, y, z)\in M^3 | x, y, z $ đôi một khác màu và $(x + y + z)\equiv 0 (mod\ 2008)\} $.
Chứng minh rằng $2|S_1| > |S_2| $.
($M^3 $ kí hiệu tích Đề các $M\times M\times M $ và $|X| $ kí hiệu số phần tử của tập hữu hạn $X $).
[RIGHT][I][B]Nguồn: MathScope.ORG[/B][/I][/RIGHT]
 
__________________
Rồng sa vũng cạn bị lươn ghẹo!
Hổ xuống đất bằng bị chó khinh!
let is offline   Trả Lời Với Trích Dẫn
Old 30-03-2008, 09:20 PM   #2
dong1919
Sư tổ Kim Dung-CÁI BANG
 
dong1919's Avatar
 
Tham gia ngày: Nov 2007
Đến từ: A1K35PBC-Nghệ An
Bài gởi: 291
Thanks: 0
Thanked 33 Times in 23 Posts
Bài số , bài đa thức và bài tổ hợp vòng 1 có vẻ dễ ( làm khá gọn )
Các bài hình chưa thử và cũng sẽ ko thử
còn bài 6 nghe bảo ko ai làm được àh
[RIGHT][I][B]Nguồn: MathScope.ORG[/B][/I][/RIGHT]
 
dong1919 is offline   Trả Lời Với Trích Dẫn
Old 30-03-2008, 09:59 PM   #3
n.t.tuan
+Thành Viên+
 
n.t.tuan's Avatar
 
Tham gia ngày: Nov 2007
Bài gởi: 1,250
Thanks: 119
Thanked 616 Times in 249 Posts
Trích:
Nguyên văn bởi dong1919 View Post
Bài số , bài đa thức và bài tổ hợp vòng 1 có vẻ dễ ( làm khá gọn )
Các bài hình chưa thử và cũng sẽ ko thử
còn bài 6 nghe bảo ko ai làm được àh
Đông làm thày phán từ bao giờ thế? Thịnh post bài rải ra các box con đi chứ?
[RIGHT][I][B]Nguồn: MathScope.ORG[/B][/I][/RIGHT]
 
__________________
T.
n.t.tuan is offline   Trả Lời Với Trích Dẫn
Old 30-03-2008, 10:05 PM   #4
psquang_pbc
+Thành Viên Danh Dự+
 
psquang_pbc's Avatar
 
Tham gia ngày: Nov 2007
Bài gởi: 747
Thanks: 9
Thanked 111 Times in 72 Posts
Gửi tin nhắn qua Yahoo chát tới psquang_pbc
Bài 2 ngày 1. Nhìn tương đối lạ, để thử xem sao. Post mấy bài khác trước vậy.
Bài 3 ngày 1 kết quả là $\lfloor \frac{2n}{3}\rfloor $. Chia tập khuyết $T $bất kì ra thành 2 phần lớn và bé thua $\frac{n}{2} $ rồi chia tiếp 2 trường hợp :
TH1. Nếu số phần tử bé thua $\frac{n}{2}\ge \frac{|T|}{2}. $.
TH2. Nếu số phần tử lớn hơn $\frac{n}{2}\ge \frac{|T|}{2}. $.
Phần dấu bằng thì chia $n $ theo $\bmod 3 $ thôi nhỉ .
Bài 1 ngày 2 thì dễ quá rồi .
Bài 3 ngày 2 hồi trước đọc sách cụ Titu cũng nhớ mang máng thế này .
Gọi tập nghiệm của phương trình $x^{2008}-1=0 $ là $S $. Khi đó thì đặt 3 đa thức đặc trưng cho mỗi màu là $P_i(x)=\sum_{a\in M_i}x^a $....Khi đó
$S_1=\frac{1}{2008}\sum_{t\in S}P_{i}(t)^3 $
$S_2=\frac{6}{2008}\prod_{t\in S}P_{i}(t) $
Và ta có dpcm. Bịa bài này cũng lời cho ai chịu khó đọc sách cụ Titu nhỉ.
Bài hình thì nhác làm.

[RIGHT][I][B]Nguồn: MathScope.ORG[/B][/I][/RIGHT]
 
__________________
[Only registered and activated users can see links. ]

No pain, no gain!

thay đổi nội dung bởi: psquang_pbc, 30-03-2008 lúc 10:08 PM
psquang_pbc is offline   Trả Lời Với Trích Dẫn
Old 30-03-2008, 10:36 PM   #5
Quân -k47DHV
+Thành Viên Danh Dự+
 
Quân -k47DHV's Avatar
 
Tham gia ngày: Jan 2008
Đến từ: Đại Học Y Hà Nội
Bài gởi: 421
Thanks: 5
Thanked 105 Times in 80 Posts
bài 1, bài 5 thì easy rồi nhưng bài 5 vẫn thích hơn cả :hornytoro:

gọi $d $là đường vuông góc với $OA $ tại$ A $ cắt $BC $ tại $A' $ khi đó tam giác$ A'AD $là tam giác cân .nên $A'L \perp AD $.$O_1 $ là trung điểm $AA' $.nên $(O_1) $ sẽ đi qua $H $( chân đường cao hạ từ $A $của tam giác $ABC $) .Tuong tự -> $H $(trực tâm tam giác $ABC $) thuộc trục đẳng phương của cả 3 đường tròn .Mặt khác cũng thấy $OA = OB = OC $.nên $P(O/(O_1) = P(O/O_2) = P(O/O_3). $
suy ra $OH $ thuộc trục đẳng phương của cả 3 đưong tròn và $G \in OH $ đpcm

b, thì $\frac{AO_1 }{AA'} = \frac1k $.dùng véc tơ để tìm đk của$ k $ sao cho $O_1 ,O_2 ,O_3 $ thẳng hàng là được
[RIGHT][I][B]Nguồn: MathScope.ORG[/B][/I][/RIGHT]
 
__________________
LƯƠNG Y KIÊM TỪ MẪU
Quân -k47DHV is offline   Trả Lời Với Trích Dẫn
Old 30-03-2008, 11:02 PM   #6
dong1919
Sư tổ Kim Dung-CÁI BANG
 
dong1919's Avatar
 
Tham gia ngày: Nov 2007
Đến từ: A1K35PBC-Nghệ An
Bài gởi: 291
Thanks: 0
Thanked 33 Times in 23 Posts
khà khà bài 4 thì ta có n lẻ ,$ 3^n+1 \vdots 2m $ là tối đa , nếu $ m \vdots 2 $và thuộc ước của $ 3^n+1 $ thì => $ (2m)^n+1 \neq 3k $ (SD căn nguyên thủy)=> dpcm
còn bài 2 thì ta áp dụng định lý đa thức là xong
bài 6 nếu có trong sách Titu có mà loạn , hồi trước nhớ thời PKH cũng có 1 bài nào đó trùng và thế là thi lại
p/s em làm thầy phán lâu lắm rồi
[RIGHT][I][B]Nguồn: MathScope.ORG[/B][/I][/RIGHT]
 

thay đổi nội dung bởi: dong1919, 31-03-2008 lúc 11:36 AM
dong1919 is offline   Trả Lời Với Trích Dẫn
Old 31-03-2008, 01:21 AM   #7
psquang_pbc
+Thành Viên Danh Dự+
 
psquang_pbc's Avatar
 
Tham gia ngày: Nov 2007
Bài gởi: 747
Thanks: 9
Thanked 111 Times in 72 Posts
Gửi tin nhắn qua Yahoo chát tới psquang_pbc
Chẳng phải trùng, trường hợp cho 3, 5 số là những bài đã được phát biểu khác đi và dạng mới rất đẹp.

Trích:
Nguyên văn bởi dong1919 View Post
khà khà bài 4 thì ta có n lẻ ,$ 3^n+1 \vdots 4 $ là tối đa , nếu $ m \vdots 4 $và thuộc ước của $ 3^n+1 $ thì => $ (2m)^n+1 \neq 3k $ => dpcm
còn bài 2 thì ta áp dụng định lý đa thức là xong
bài 6 nếu có trong sách Titu có mà loạn , hồi trước nhớ thời PKH cũng có 1 bài nào đó trùng và thế là thi lại
p/s em làm thầy phán lâu lắm rồi
Định lý gì ? Lại chém gió rồi :hornytoro:.
[RIGHT][I][B]Nguồn: MathScope.ORG[/B][/I][/RIGHT]
 
__________________
[Only registered and activated users can see links. ]

No pain, no gain!
psquang_pbc is offline   Trả Lời Với Trích Dẫn
Old 31-03-2008, 11:36 AM   #8
dong1919
Sư tổ Kim Dung-CÁI BANG
 
dong1919's Avatar
 
Tham gia ngày: Nov 2007
Đến từ: A1K35PBC-Nghệ An
Bài gởi: 291
Thanks: 0
Thanked 33 Times in 23 Posts
Hơ định lý cơ bản về đa thức
định lý này chắc ai cũng biết chứ
p/s nếu ko trùng thì sao phải thi lại nhỉ
khà khà lại thêm 1 bài
[RIGHT][I][B]Nguồn: MathScope.ORG[/B][/I][/RIGHT]
 
dong1919 is offline   Trả Lời Với Trích Dẫn
Old 31-03-2008, 12:17 PM   #9
psquang_pbc
+Thành Viên Danh Dự+
 
psquang_pbc's Avatar
 
Tham gia ngày: Nov 2007
Bài gởi: 747
Thanks: 9
Thanked 111 Times in 72 Posts
Gửi tin nhắn qua Yahoo chát tới psquang_pbc
Làm gì trùng . Số nó khác đi mà, đâu tính trùng được .
[RIGHT][I][B]Nguồn: MathScope.ORG[/B][/I][/RIGHT]
 
__________________
[Only registered and activated users can see links. ]

No pain, no gain!
psquang_pbc is offline   Trả Lời Với Trích Dẫn
Old 31-03-2008, 07:46 PM   #10
vănđhkh
+Thành Viên Danh Dự+
 
vănđhkh's Avatar
 
Tham gia ngày: Nov 2007
Đến từ: Huế-Quảng Bình
Bài gởi: 74
Thanks: 6
Thanked 67 Times in 19 Posts
Gửi tin nhắn qua Yahoo chát tới vănđhkh
Năm ngoái 1 bài hình thi VMO đã dùng trục đẳng phương và đến năm nay,thi TST cũng dùng đến trục đẳng phương,không chừng năm sau IMO sẽ dùng trục đẳng phương cũng nên --> Phán chẳng khác gì ku Đông
[RIGHT][I][B]Nguồn: MathScope.ORG[/B][/I][/RIGHT]
 
__________________
Thành Văn™_vtv
vănđhkh is offline   Trả Lời Với Trích Dẫn
Old 31-03-2008, 10:34 PM   #11
dong1919
Sư tổ Kim Dung-CÁI BANG
 
dong1919's Avatar
 
Tham gia ngày: Nov 2007
Đến từ: A1K35PBC-Nghệ An
Bài gởi: 291
Thanks: 0
Thanked 33 Times in 23 Posts
Năm sau thi ĐH ko khéo cũng xài trục đẳng phương đó , anh em cẩn thận nhé
[RIGHT][I][B]Nguồn: MathScope.ORG[/B][/I][/RIGHT]
 
dong1919 is offline   Trả Lời Với Trích Dẫn
Old 01-04-2008, 09:44 PM   #12
math man
+Thành Viên+
 
Tham gia ngày: Mar 2008
Bài gởi: 23
Thanks: 0
Thanked 3 Times in 3 Posts
Nhận xét thử

đề thi này em thấy dễ nhất là bài đa thức
kết quả là n=1.
Khi đó R(a,b)=b
P(x)=$\ x^2 $
Q(x)=x
bài này chỉ cần xét khi n>1 vàn=1 là làm được ngay
[RIGHT][I][B]Nguồn: MathScope.ORG[/B][/I][/RIGHT]
 
math man is offline   Trả Lời Với Trích Dẫn
Trả lời Gởi Ðề Tài Mới

Bookmarks


Quuyền Hạn Của Bạn
You may not post new threads
You may not post replies
You may not post attachments
You may not edit your posts

BB code is Mở
Smilies đang Mở
[IMG] đang Mở
HTML đang Tắt

Chuyển đến


Múi giờ GMT. Hiện tại là 05:38 PM.


Powered by: vBulletin Copyright ©2000-2024, Jelsoft Enterprises Ltd.
Inactive Reminders By mathscope.org
[page compression: 87.42 k/100.57 k (13.08%)]